Teorema de Stone-von Neumann

De acuerdo con el teorema de Stone-von Neumann , cualquiera de los dos operadores autoadjuntos conjugados canónicamente siguiendo la relación:

[ q ^ , pag ^ ] = i
no pueden ser ambos acotados. Estoy confundido acerca de cómo probamos esta parte y qué significa físicamente. ¿Alguien puede explicar?

No, no es una consecuencia del teorema de Stone-von Neumann que los dos operadores (o al menos uno) deban ser ilimitados, es una consecuencia del teorema de Wielandt-Wintner.

Respuestas (3)

Comenté que el teorema de Stone-von Neumann no es una prueba para la declaración al comienzo de la pregunta. Las demostraciones originales del teorema de Wielandt-Wintner (probablemente solo en 1947-1948, mientras que el teorema de Stone-von Neumann ya tenía una demostración satisfactoria de von Neumann en 1931) se encuentran en:

Wintner, A. - The Unboundness of Quantum-Mechanical Matrices (1947, The Physical Review, Vol. 71, p. 738-739)

Wielandt, H. - Über die Unbeschränktheit der Operatoren der Quantenmechanik (1948, Mathematische Annalen, p. 21).

La esencia de la prueba de Wielandt es la nota 6 de la página Wiki citada:

ingrese la descripción de la imagen aquí

La importancia de tener operadores ilimitados de coordenadas y cantidad de movimiento en el eje real (1D) es que el "movimiento cuántico" de la partícula no está restringido, en el sentido de que la coordenada o la cantidad de movimiento pueden medirse en un valor alto arbitrario (infinito en el sentido opuesto). límite), es decir, matemáticamente, los operadores ilimitados no tienen un espectro limitado.

Bien, pero ¿qué sucede si aplicamos el mismo enfoque a la relación de incertidumbre energía-tiempo, ya que podemos encontrar la energía? Decimos que el operador de tiempo no existe ya que el hamiltoniano está acotado por abajo, pero ¿por qué?
Este es el tema de uno de los mitos de QM, el llamado teorema de Pauli. Volveré sobre este tema. Nikolic lo aborda superficialmente aquí: arxiv.org/abs/quant-ph/0609163
Francamente señor, algunos puntos en el documento dado no parecen correctos, creo que es bastante poco confiable porque trata de presentar todo con certeza.
Sé que es parcialmente incorrecto pero Found.Phys. tiene un proceso de revisión por pares débil, por lo que a veces incluso se publica basura.
Sus preguntas complementarias en los comentarios se pueden vincular a esta pregunta: physics.stackexchange.com/q/376822 . Comprueba mi respuesta allí.

Teorema: si dos operadores acotados (no necesariamente autoadjuntos) q ^ y pag ^ en un espacio de Hilbert satisfacer el CCR

(1) [ q ^ , pag ^ ]   =   i   1 , i     C ,
entonces i = 0 .

Prueba indirecta: (Esta es esencialmente la prueba de la Ref. 1.) Suponga

(2) i     0.
Desde pag ^ está acotado, podemos cambiar
(3) pag ^   :=   pag ^ + b 1
por una cantidad positiva finita b > 0 , de modo que pag ^ es un operador invertible, por lo que pag ^ y pag ^ 1 ambos son operadores acotados. Nótese que el operador primado pag ^ también cumple con el CCR (1). Dejemos la notación prima de ahora en adelante. los espectros
(4) σ ( q ^ pag ^ )   =   σ ( pag ^ q ^ pag ^ pag ^ 1 )   =   σ ( pag ^ q ^ )
de los operadores acotados q ^ pag ^ y pag ^ q ^ deben ser conjuntos acotados iguales. Por otro lado, el CCR (1) muestra que los espectros están desplazados
(5) σ ( q ^ pag ^ )   = ( 1 )   σ ( pag ^ q ^ ) + i
La única forma en que las ecs. (2), (4) y (5) podrían no ser mutuamente contradictorios si los espectros son los conjuntos vacíos. Sin embargo, esto contradice el hecho general (mencionado, por ejemplo, en Wikipedia y MO.SE ) de que

Realidad: Todo operador acotado tiene un espectro no vacío.

Observación: Si además suponemos que q ^ y pag ^ son autoadjuntos, no necesitamos usar el hecho anterior. Entonces el conmutador (1) es anti-autoadjunto, de modo que R debe ser real Además el operador acotado

(6) s ^   :=   q ^ pag ^ i 2   = ( 1 )   pag ^ q ^ + i 2   =   s ^ ,
es autoadjunto, y por lo tanto (del teorema espectral ) tiene un espectro real no vacío
(7)     σ ( s ^ )   = ( 6 )   σ ( q ^ pag ^ ) i 2   = ( 1 )   σ ( pag ^ q ^ ) + i 2 ,
que es la ec. (5) sin la escapatoria de los conjuntos vacíos.

Referencias:

  1. A. Winterner, Phys. Rev. 71 (1947) 738 .
¿Puede decirme cuál habría sido el resultado cuando se aplicó el operador en el estado traducido si de hecho estuviera acotado?
Señor, soy un estudiante universitario que acaba de comenzar a estudiar mecánica cuántica, y todavía estoy confundido sobre cómo podemos decir a partir de esto que el operador no debe estar acotado.
Esta prueba es incorrecta en muchos niveles. No se puede elegir un estado propio normalizado. No se garantiza que esto exista, solo porque el operador es autoadjunto y está acotado (el operador de coordenadas para la partícula en un cuadro finito es un ejemplo). (5) no tiene sentido, porque el operador norma es el supremo de todos los q's, ¡así que q' no puede ser mayor que sí mismo! En realidad, (3) está mal, porque la norma de (por existir absurdo) | q ) es igual a la norma de | q , porque la exponencial compleja de un operador autoadjunto es un operador isométrico!!
Ups, el teorema en el que estoy pensando parece funcionar solo con la suposición adicional de que el operador es compacto. Respuesta actualizada con nueva prueba.
@Qmecanico. La nueva prueba también es incorrecta. ¿De dónde viene (4)? La primera igualdad es verdadera, si y sólo si p es unitaria (acotada e isométrica), pero se supuso que p era autoadjunta. El único operador unitario y autoadjunto es el operador unitario. Si pag ^ = 1 ^ , entonces el CCR no tiene sentido.

Las variables/operadores conjugados están relacionados por la transformada de Fourier, es decir, los estados (cuánticos) de un observable son la transformada de Fourier del otro y, como tal, solo uno de ellos puede tener un soporte compacto (a menos que sea una función cero). Esto se conoce como la relación de Incertidumbre en las transformadas de Fourier . Intuitivamente, significa que la dispersión de una variable y su dual de Fourier son inversamente proporcionales, lo que se traduce físicamente, por ejemplo, en que la posición se localiza (concentra) y el impulso se deslocaliza (dispersa). Para un enfoque de prueba, vea la respuesta de Qmechanic.

Físicamente, todos estos tipos de variables/observables son incompatibles (no conmutables). X PAG PAG X 0 , dónde PAG F 1 X F con F : L 2 ( R ) L 2 ( R ) ), ya que no se pueden medir simultáneamente con una precisión arbitraria. En otras palabras, las incertidumbres en las dos variables siempre están limitadas por el promedio de su conmutador (incluso si realizó las mediciones por separado en un conjunto de infinitos sistemas cuánticos preparados de manera idéntica). Estas incertidumbres son una propiedad intrínseca de cualquier estado cuántico.

¿Qué es la transformada de Fourier de un operador? El operador Exp ( X 2 ) está ligado. ¿Cuál es su FT? Lo es Exp ( PAG 2 ) ? Si es así, también está acotado, lo que contradice su afirmación.
¿Puedes explicar esto en términos de energía y tiempo, ya que también son canónicamente conjugados, también siguen un principio de incertidumbre, pero la energía se puede calcular?
@HarshdeepSingh ese es un tipo algo especial de relación de incertidumbre, pero está bien discutido en física SE, consulte, por ejemplo,: physics.stackexchange.com/questions/53802/…
@AccidentalFourierTransform Por supuesto, F T 's un operador unitario y si conjuga cualquier operador acotado con él, debe permanecer acotado. Decir para posición y momento, operador de posición conjugado con F , es decir pag F 1 X F con F aquí el L 2 FT normalizado, le da el operador de impulso. Por otro lado, por lo que respecta a la relación de incertidumbre en QM, digamos nuevamente para la posición y el momento, la transformada de Fourier se aplica a los estados cuánticos mismos (y luego sigue la discusión sobre la compacidad), la incertidumbre en términos de operadores se expresa como el CCR.
@AccidentalFourierTransform Además, reformulé ligeramente las cosas para evitar confusiones.